if pCm then which of the following set represent pN (PUM) ?
Answers
Answered by
3
Answer:
iven:
P\,{\subset}\,MP⊂M
\textbf{To find:}To find:
P\,{\cap}(P{\cup}M)P∩(P∪M)
\textbf{Solution:}Solution:
\text{We know that,}We know that,
\bf\,A,B\;\textbf{and}\;C\;\textbf{are any three sets}A,BandCare any three sets
\textbf{Then,}Then,
\bf(i)\,A{\cap}(B{\cup}C)=(A{\cap}B){\cup}(A{\cap}C)(i)A∩(B∪C)=(A∩B)∪(A∩C)
\bf(ii)\textbf{If $\bf\,A\,{\subset}\,B,$ then $\bf\,A{\cap}B=A$}(ii)If A⊂B, then A∩B=A
\bf(iii)\textbf{For any set $\bf\,A,\;\;$$\bf\,A{\cap}A=A$}(iii)For any set A,A∩A=A
\text{Consider,}Consider,
P\,{\cap}\,(P\,{\cup}\,M)P∩(P∪M)
=(P\,{\cap}\,P)\,{\cup}\,(P\,{\cap}\,M)=(P∩P)∪(P∩M)
=P{\cup}P=P∪P
=P=P
\textbf{Answer:}Answer:
\boxed{\bf\,P\,{\cap}(P{\cup}M)=P}
P∩(P∪M)=P
Answered by
1
right answer is
mark me as
Attachments:
Similar questions